Pessoal,
Tô realmente apanhando um bocado da indução finita... Não consigo enxergar
as manipulações necessárias para provar essas coisas.
Alguém poderia me dar uma ajuda com esse?

Provar que n! > n^3, pra n >= 6.

Grato,
Henrique.

=========================================================================
Instruções para entrar na lista, sair da lista e usar a lista em
http://www.mat.puc-rio.br/~nicolau/olimp/obm-l.html
=========================================================================

Responder a